Which of the following are solutions to the equation below?
Check all that apply.
x^2 + 4x-9 = 5x + 3

Which Of The Following Are Solutions To The Equation Below?Check All That Apply.x^2 + 4x-9 = 5x + 3

Answers

Answer 1

Answer:

Answer:

x=−3 & x=4

Step-by-step explanation:

Step 1: Subtract 5x+3 from both sides.

x2+4x−9−(5x+3)=5x+3−(5x+3)

x2−x−12=0

Step 2: Factor left side of equation.

(x+3)(x−4)=0

Step 3: Set factors equal to 0.

x+3=0 or x−4=0

x=−3 or x=4

Answer 2

Answer:

x=4  x=-3

Step-by-step explanation:

x^2 + 4x-9 = 5x + 3

Subtract 5x from each side

x^2 + 4x-5x-9 = 5x -5x+ 3

x^2 +-x-9 =  3

Subtract 3 from each side

x^2 -x-9-3 =  3-3

x^2 -x -12 =0

Factor

What 2 numbers multiply to -12 and add to -4

-4*3 = -12

-4+3 =-1

(x-4)(x+3) =0

Using the zero product property

x-4 =0  x+3 =0

x=4  x=-3


Related Questions

Plz help guys, it is a pretty easy sum​

Answers

Answer:

a3+13a

Step-by-step explanation:

sorry I don't know how to solve this

Answer:+-3

Explanation:

I added 4 on both side so that a^2+1/a^2-2 becomes a^2+1/a^2+2 which is Special products of (a^2+1/a^2).

With the given information, which of these conclusions is reasonable? A.Town A and Town B had the same low temperatures. B.The temperatures in each town were the same every day. C.The mean low temperatures in Town A and Town B were the same. D.The variability of temperatures in Town A and Town B was the same.

Answers

Answer:

A. Town A and Town B had the same low temperatures.

Step-by-step explanation:

The towns both A and B had low temperatures is shown by the same rate of low temperatures.

Which is the best estimate of -14 1/9 (-2 9/10)

Answers

Answer:

Step-by-step explanation:

-14 1/9 is close to - 14

-2 and 9/10 is close to - 3

The best estimate would be 42 (14 * 3)

Estimate means you put your calculator on the kitchen counter until you've done this question.

Let's see what the actual answer is. 40.92222 which is close to 41.

42 is a pretty good estimate.

Find the least number by which the following number must be multiplied so that the product are perfect cube one number 72 to number 128 number 288 phone number 675​

Answers

Answer:

Well this question is actually a piece of cake. Just pick your favorite number. Multiply it by 10. Then do whatever operation you want with the 2,300. For the exponent part of this. Lets say we do it this way y times z equals 2,300. Exponents are letters used in mathematical terms. So any letter can be used to represent any number.

Step-by-step explanation:

Based only on the information given in the diagram, which congruence
theorems or postulates could be given as reasons why AABC= ALMN?
Check all that apply
O A. LL
O B. ASA
I C. LA
D. HL
E AAS

Answers

3 Answers:

Choice A.  LLChoice D.  HLChoice F.  SAS

==========================================================

Explanation:

Let's go through the answer choices one by one.

A) This can be used because LL = leg leg, and this means we have two pairs of congruent legs. Those pairs are AC = LN and CB = NM. The LL theorem only applies to right triangles.B) This cannot be used. We don't have info about two pairs of angles. We only know that one pair of angles are the same (those 90 degree angles). So we can't form the second "A" in "ASA". This idea will come up again in choice C and choice E.C) This cannot be used. Why not? Because the "A" of "LA" refers to "acute angle". But unfortunately we don't know anything about the acute angles (whether they are congruent or not). The LA theorem can only be applied to right triangles.D) This can be used. We can use the HL (hypotenuse leg) theorem because we see that AB = LM are the pair of congruent hypotenuses, and you can use any of the congruent leg pairs to form the L of HL. Similar to LL and LA, the HL theorem only works for right triangles.E) This cannot be used. Like with choice B, we can't form the second "A" of "AAS".F) This can be used because we have two pairs of congruent sides, with a pair of congruent angles between those sides. Those angles being the marked 90 degree angles. It turns out that LL theorem is a special case of the SAS theorem.

In short, we can use choice A, choice D, choice F. We can't use the other three choices because we lack the info about any other pairs of angles.

The congruence theorem or postulate that we can use to show that triangle ABC is congruent to triangle LMN is LL (Side-Side-Side), the correct option is A.

What are congruent triangles?

Suppose it is given that two triangles ΔABC ≅ ΔDEF

Then that means ΔABC and ΔDEF are congruent. Congruent triangles are exact same triangles, but they might be placed at different positions.

The order in which the congruency is written matters.

For ΔABC ≅ ΔDEF, we have all of their corresponding elements like angle and sides congruent.

Thus, we get:

[tex]\rm m\angle A = m\angle D \: or \: \: \angle A \cong \angle D \angle B = \angle E\\\\\rm m\angle B = m\angle E \: or \: \: \angle B \cong \angle E \\\\\rm m\angle C = m\angle F \: or \: \: \angle C \cong \angle F \\\\\rm |AB| = |DE| \: \: or \: \: AB \cong DE\\\\\rm |AC| = |DF| \: \: or \: \: AC \cong DF\\\\\rm |BC| = |EF| \: \: or \: \: BC \cong EF[/tex]

(|AB| denotes length of line segment AB, and so on for others).

We are given that;

Sides are equal

Now,

Based only on the information given in the diagram, we can use the following congruence theorems or postulates to show that triangle ABC is congruent to triangle LMN:

A. LL (Side-Side-Side): This theorem states that if three sides of one triangle are congruent to three sides of another triangle, then the triangles are congruent. In this case, we know that AB = LM, AC = LN, and BC = MN, so we can use LL to show that triangle ABC is congruent to triangle LMN.

B. ASA (Angle-Side-Angle): This theorem states that if two angles and the included side of one triangle are congruent to two angles and the included side of another triangle, then the triangles are congruent. In this case, we do not know any angle measures, so we cannot use ASA to show that the triangles are congruent.

Therefore, by the  congruent triangles the answer will be LL (Side-Side-Side).

Learn more about congruent triangles here:

https://brainly.com/question/16921692

#SPJ7

F(x) = 3x + 2 what if f(5)

Answers

Ans is 17 . Solved it Down below. Hope it help

What is the volume of the cylinder?
•576 cm3
•2887 cm3
•96 cm3
•192 cm3

Answers

[tex] \large\begin{gathered} {\underline{\boxed{ \rm {\red{Volume \: \: of \: \: cylinder \: = \: \pi \: {r}^{2} \: h }}}}}\end{gathered}[/tex]

r represents radius of cylinder.

h represents height of cylinder.

So ,

r = 6 cm

h = 16 cm

π = 3.14

Substuting the values

⇥Volume of the cylinder = π r² h

⇥Volume of the cylinder = 3.14 × 36 × 16

⇥Volume of the cylinder = 113.04 × 16

⇥Volume of the cylinder = 1808.64

Hence , the volume of cylinder is 1808.64 cm²

please help in indices
[tex] \frac{ {5}^{m + 2} - {5}^{m} }{ {5}^{m + 1} + {5}^{m} } \\ \\ \frac{ {4}^{m} + {4}^{m + 1} }{ {4}^{m + 2} - {4}^{m} } [/tex]

Answers

Step-by-step explanation:

Hey there!

Please see your required answer in picture.

Hope it helps!

Please find attached photograph for your answer.

Hope it helps.

Do comment if you have any query.

Problem 3 Find the value of x.​

Answers

value of x should be 3.2 units

Answered by Gauthmath must click thanks and mark brainliest

3.2 units in your answer

ILL MARK BRAINIEST IF YOU DO THIS CORRECTLY!!!

Answers

Hmmm im on the same question so i think its this

5 a + 3 b + 2 a b + 4 a -7 b​

Answers

Answer:

5a+4a=9a

3b-7b=-4b

2ab

=9a-4b+2ab or 2ab+9a-4a

Step-by-step explanation:

Calculate liked terms

how many weeks are there in 504 hours

Answers

Answer:

26297.4 weeks

Step-by-step explanation:

hope it will help u

please mark me brillient

quadrilateral abcd is symmetric with respect to the y axis. the coordinates of point a are (-2,2), and the coordinates of point c are (2, 1). if b is in the first quadrant, what are the coordinates of b?​

Answers

Answer:

The coordinate of b = (2, 2)

Step-by-step explanation:

The details of the quadrilateral abcd are;

The quadrilateral is symmetric about the y-axis (the line x = 0)

The coordinates of the vertices are; a(-2, 2), c(2, 1)

The location of the vertex, b = The first quadrant

We have;

The line [tex]\overline {ab}[/tex] is perpendicular to the line [tex]\overline {bc}[/tex]

Let (x, y) represent the coordinate of the vertex b, we have;

(y - 2)/(x - (-2)) = (y - 2)/(x + 2) = -1/(y - 1)/(x - 2) = - (x - 2)/(y - 1)

(y - 2)·(y - 1) = -(x + 2)·(x - 2)

y² + x² - 3·y - 2 = 0...(1)

y² + x² = 3·y + 2

Also we have;

(y - 2)² + (x - 1)² + (y - 2)² + (x - (-2))² = (2 - 1)² + (2 - (-2))² = 17

Therefore;

2·y² + 2·x² - 8·y + 2·x + 13 = 17

2·(3·y + 2) - 8·y + 2·x + 13 = 17

Using an online tool, we have;

x = y

From equation (1), we have;

2·y² - 3·y - 2 = 0

∴ y = 2, or y = -1/2

Where y = 2, we have;

x = y = 2

Therefore, the point b = (2, 2).

solve A = 1/2bh, for b

Answers

Answer:

b=2A/h

Step-by-step explanation:

First I throw everything else to another side with A by dividing them

(A = b)/2h

then I get

b=2A/h

Step-by-step explanation:

A = 1/2 bh

divide by 1/2

2A = bh

divide by h

2A/h = b

Plz solve question 12

Answers

Answer: C

Step-by-step explanation:

To solve for 12, we can use eliminate or substituion to solve our system of equations. Let's use elimination method.

[tex]\left \{ {{3f-2k=10} \atop {-3f-2k=14}} \right.[/tex]

Let's add the equations together. This way, 3f+(-3f)=0

[tex]-4k=24[/tex]                   [divide both sides by -4]

[tex]k=-6[/tex]

Now that we know k, we can plug it into either equation to find f.

[tex]3f-2(-6)=10[/tex]        [multiply]

[tex]3f+12=10[/tex]             [subtract both sides by 12]

[tex]3f=-2[/tex]                    [divide both sides by 3]

[tex]f=-\frac{2}{3}[/tex]

Now that we have f and k, we know that C is the correct answer.

For g(x) = x - 2 find the value of x for which g(x) = -4.

Answers

Answer:

x=-2

Step-by-step explanation:

g(x) = x-2

Let g(x) = -4

-4 =x-2

Add 2 to each side

-4+2 = x-2+2

-2 = x

Answer:

x = -2

Step-by-step explanation:

g(x) = -4

g(x) = x - 2

-4 = x - 2

+2      +2

-2 = -x

Please help me find which expression is correct

Answers

Answer:

IN MY OPINION D NO IS THE CORRECT ANSWER OF YOUR QUESTION.

Answer:

in my opinion d is the correct answer of your questions.

Step-by-step explanation:

Because subtract subtract sign is always add.

hope this will help you

thanks

I have 5 digits.My eight is worse 8000. One of my sixes is worth 60.The other is worth 10 times as much.My other digit is a zero

Answers

Answer:

i think its 58,660

Step-by-step explanation:

Answer:

58,660

Step-by-step explanation:

Explain the process due to which rain falls ? Class 4 - EVS​

Answers

Answer:

As you may already know, water drops that fall from the cloud are considered "rain".

The Sun's heat turns the moisture or water from leaves, plants, rivers, lakes, and oceans - and turns it into gas or also called, vapor. This water vapor then turns into gas and disappears into the air. When it gets mixed with the air, it cools down. When it cools down, it changes into small water drops, which then form a cloud. These small water drops join together with other water drops to create larger and bigger water drops.

You may know this now because this is the easiest part. What happens when something gets heavy or is over-filled? It falls down, right?

So, the large drops from water fall down as they get too heavy for the cloud to carry. These big droplets falling down on us are called Rain.

A. 4√29/ 7 feet
b. 4√5 feet
c.√164 feet
d. 2√41

Answers

Answer:

These two triangles are similar triangles. This means that their side lengths are proportional to each other.

Thus, making line segment EC equal to "x", and BC equal to "y" we can write:

8/y = 28/(10+y)

The next step is to get rid of the fractions, which can be done by cross multiplying.

So we have:

8(10+y) = 28(y)

After distribution and some simplification, you should get the value of y.

80+8y = 28y

80 = 20y

80/20 = 20y/20

4 = y

y = 4

Knowing that y = BC, and y = 4, it is clear that BC = 4.

Since BC = 4, one can use the Pythagorean Theorem to solve for segment EC.

Pythagorean Theorem: a^2 + b^2 = c^2, where a and b are the side lengths of a right triangle, and c is the hypotenuse (in other words the longest side)

In our case, a and  b are 8 and 4 (the order doesn't really matter here).

So we have: 8^2 + 4^2 = c^2

64 + 16 = c^2

80 = c^2

c = sqrt 80

c  = 4 sqrt 5

And we arrive at the answer- EC = 4 sqrt 5, making B the correct choice.

Hope this helps!

Please help me with this anyone

Answers

Answer:

Step-by-step explanation:

Begin by combining like terms and then factoring. Combining like terms will give you

[tex]10p^2-17p-20=0[/tex] Using the "old-fashioned" way of factoring, the a times c method, our a = 10, b = -17 and c = -20.

a * c = 10(-20) = -200 and now we need the factors of 200 (don't worry about the negative) that combine to give us that middle term, -17p (here is where the negative matters). The factors of 200 are:

1. 200;  2, 100;  4. 50;  5, 40;  8, 25;  10, 20

The combination of those numbers that can be manipulated to give us a -17p is the 8, 25 as long as we say that the 25 is negative and the 8 is positive. Rewrite the original polynomial to reflect those factors:

[tex]10p^2-25p+8p-20=0[/tex] and then factor by grouping:

[tex](10p^2-25p)+(8p-20)=0[/tex] and factor out from each set of parenthesis what is common:

[tex]5p(2p-5)+4(2p-5)=0[/tex] again factor out what is common:

(2p - 5)(5p+ 4) = 0. These are the factors; therefore the solutions are

2p - 5 = 0 so

2p = 5 and

p = 5/2  and

5p + 4 = 0 and

5p = -4 so

p = -4/5

Will Mark Brainnlest Please help me​

Answers

Answer:

sory

Step-by-step explanation:

soryuuuiuuuiiiiiiijjjjkkkkkj

The answer is: 9m-n+6

HELP!!

Resolve into factors:

8 – 6а – 9a^2 + 27a^3​

Answers

[tex]\\ \sf\longmapsto 8-6a-9a^2+27a^3[/tex]

[tex]\\ \sf\longmapsto 27a^30-9a^2-6a+8[/tex]

Take 3a common

[tex]\\ \sf\longmapsto 3a(9a^2-3a-2)+8[/tex]

Hence done

can someone help me with this one ....​

Answers

Answer:

-5, - 2, 3

Step-by-step explanation:

y=2x+3, y=-7, x=-5; y=-1, x=-2, y=9, x=3

what is the gcf of 36, 126, and 210?

Answers

Answer:

Greatest common factor (GCF) of 36 and 210 is 6.

Step-by-step explanation:

Answer:

6

Brainliest, please! (Almost an Ace!)

Step-by-step explanation:

Look at the smallest factor, 36. Find its factors.

36: 1, 2, 3, 6, 12, 18, 36

Out of all of them, which is the biggest one that 126 and 210 are also divisible by?

We see that they're divisible by 1, 2, 3, and 6.

Our answer is 6.

Use the Diagram to find BC.

Answers

Answer:

Step-by-step explanation: you have to ask your parents for help

(4a)^2 without the exponents

Answers

Answer:

16 a*a

Step-by-step explanation:

(4a)^2

(4a) (4a)

16 a*a

Answer:

16a²

Step-by-step explanation:

(4a)²

=>(4)² x (a)²

=> 16 x a²

=>16a²

An equation is shown below:
3(2x – 7) = 3

Part A: How many solutions does this equation have? (4 points)

Part B: What are the solutions to this equation? Show your work. (6 points)

Answers

Answer:

Part A: One | Part B: x = 4

Step-by-step explanation:

[tex]3(2x-7)=3\\6x-21=3\\6x=24\\x=4[/tex]

Answer:

4

Step-by-step explanation:

3(2x _7)=3

multiply 3 by everything in the bracket

6x-21 = 3

move 21 to the other side of the equation,it becomes positive sine it crosses the = sign

6x = 3+21

6x =24

divide both sides by 6

6x/6=24/6

x = 4

Find the surface area of each figure. Round your answers to the nearest tenth, if necessary​

Answers

Answer: 109m^2

Step-by-step explanation:

(5)(8.4)/2 = 21 m^2

(4 ) (21) = 84 m^2

Base = 5^2 = 25 m^2

84 + 25 = 109 m^2

Find the arc length of the partial circle.

Answers

Answer: C=10.99

Step-by-step explanation:

It can be seen that the arc has an angle of a=90 degrees, respectively, the length of the arc can be found by the formula[tex]\bf C=\dfrac{2\bf {\pi} r}{360} \cdot \alpha=> \dfrac{2\cdot 3,14\cdot 7 }{360} \cdot 90=\boxed{\bf 10.99}[/tex]  
Other Questions
In the following scenario for a hypothesis test for a population? mean, decide whether the? z-test is an appropriate method for conducting the hypothesis test. Assume that the population standard deviation is known. Preliminary data analyses reveal that the sample data contain no outliers but that the distribution of the variable under consideration is probably mildly skewed. The sample size is 70.Choose the correct answer below.a. The z-test is not an appropriate method, because the sample size is too small to be useful. b. The z-test is an appropriate method, because the sample contains no outliers. c. The z-test is an appropriate method, because the sample size is sufficiently large that the skewness of the variable does not matter. d. The Z-test is not an appropriate method, because the sample is not a large sample and the data are highly skewed. What is the slope of the line that goes through (-1, 7) and (3, 9)? Write a paragraph on online classes in 150 words.(for stander 6,7) Fast-forwardingtomoderntimes,theheydayofthehuman-poweredflightisfarfromover.Lucrativeprizesstillaboundforrotor-drivencraft,whilstexperimentalversionsofhelicoptersandornithopters(flyingmachineswithflapping,bird-likewings)arebothonthedrawingboardandleapingoffitwithgreatabandon.Thedreamofhuman-poweredflighthasbeenanofficialrealitynowformorethan50years,butthelimitsoftheiringenuityareboundlessespeciallywheninventorsandengineersarespurredonbyprizesaslargeasthe$250,000onofferfromtheAmericanHelicopterSocietysIgorSikorskyPrizeFund.Andwhoknows,perhapsonedayhiscommutetoworkwilltakeplacenotonabicyclebutonabicycle-poweredcraft.Doesyourofficehavealandingpadontherooforarunwaynearby?Thedaywhenshedoesmightjustbecloserthantheythink!what are the four pronoun-related errors in this paragraph? Please answer this question. Will give brainiest fast The theft of currency over a long period of time that tends to be commissioned for a modest amount of money but may not be considered to be financially damaging is calledATM burglaries.embezzlement.forgery.credit card fraud. Q is equidistant from the sides of TSR. Find the value of x.T(2x + 24030SR Which two factors does the power of a machine depend on? . work and distance B.. force and distance C. work and time D. time and distance? The risk-free rate of return is 5%, the required rate of return on the market is 15%, and High-Flyer stock has a beta coefficient of 1.4. If the dividend per share expected during the coming year, D1, is $3.92 and = 5%, at what price should a share sell? Which among the following expresses Mr. Dimmesdales guilt?Trusting no man as his friend, he could not recognize his enemy when the latter actually appeared.Speak thou for me! cried she. Thou wast my pastor, and hadst charge of my soul, and knowest me better than these men can.Come up hither, Hester, thou and little PearlYe have both been here before, but I was not with you. Come up hither once again, and we will stand all three together!Death was too definite an object to be wished for or avoided. gegygwugduwbudbwbdubwudbuwh7fh7whf8hw8hf8hw8hf8e Anthony was interviewing an applicant for the position of volunteer guide at an art museum. During the course of the interview, the applicant began to discuss her interest in introducing young children to art. Based on how the conversation progressed, Anthony followed up with several questions about how young children react to art, why she felt early exposure to art was important, and how she would design situations for children to experience art. Anthony was most likely conducting a Why does the GI tract have a plexus in the muscalaris and nerves in the mucosa? Choose the word that correctly completes the sentence.Tu_quinze ans.asai,aveza Do a little research on your community's Web sites to find out what kind of conservation or preservation measures have been accomplished and the effect they have had on the community. When you have completed this task, write a 400-word paper that outlines what has been done and what is to be done in the future. Determine the type of quadrilateral given the following coordinates. Show and explain all steps to prove your answer. A(2, 3) B(-1, 4) C(0, 2) D(-3, 3) Refer to the picture above What is the correct order for the reactions that produce the following transformation. a. (1) H2/Lindlar (2) CH3CO2OH b. (1) H2/Lindlar (2) O3, Zn, HCl c. (1) H2/Pd (2) CH3CO2OH d. (1) Na, NH3 (2) CH3CO2OH Type the correct answer in the box. Use numerals instead of words. If necessary, use / for the fraction bar.If x+2 is a factor of x^3-6x^2-11x+k then k= Consider the scenario and use complete sentences to answer the question.Two players each roll an ordinary number cube. The sum is found of the two numbers showing. If the sum is an even number, player A gets 1 point. If the sum is an odd number, player B gets 1 point. The game ends after 6 rounds.Is this a fair game? Use the possible results of the game to support your answer.